Using the similar triangles, which equation could be used to find the slope of line AB? A) m = p q B) m = q p C) m = n m D) m = q n

Answers

Answer 1
Your answer is A



Mark brainliest!

Related Questions

If h(x) = x2-5x + 7
find: h(-5)

Answers

Answer:

h(-5)=22

Step-by-step explanation:

Replace x with -5.

[tex]h(-5) = (-5)2-5(-5) + 7[/tex]

Simplify.

[tex]h(-5)=-10+25+7[/tex]

Simplify.

[tex]h(-5)=22[/tex]

It is −3)=57. Explanation: Hence h(x)=5x2−4x then just replace x for −3. h(−3)=5⋅(−3)2−4⋅(−3)⇒h(−3)=45+12⇒h(−3)=57

Halfway through the second quarter, what fructional part of the game is left?

Answers

Answer:

5/8 of the game

Step-by-step explanation:

Half of a quarter is 1/2, and 1 quarter is already done, therefore 1/2, or to simplify it 1/8, plus 2/8, would be 3/8. Now, a quarter is 1/4th of something therefore there must be 4 quarters in a total.so that means that there are 8 halfves  in total , so 8-3=5, so the fractional parts of the game left is 5/8.

The part of the game i.e. left should be [tex]5\div 8[/tex]

Calculation of the part of the game:

Since the half of the quarter is one-half. And, the one quarter is done so one-half is remaining. That means

[tex]= 1\div 8 + 1\\\= 2\div 8[/tex]

Since one-fourth should be of something also there are total 4 quarters that means there are 8 halves.

So, remaining it should be

= 8 -3

= 5

Hence, The part of the game i.e. left should be [tex]5\div 8[/tex]

Learn more about game here: https://brainly.com/question/1415899

what number would you subtract from each side of inquality to solve y+3/8>16​

Answers

Answer: I would subtract 3/8 from both sides

Explanation:

y + 3/8 > 16
y > 16 - 3/8 (subtract 3/8 from both side)
y > 128/8 - 3/8
y > 125/8

Answer:

[tex]y > 125[/tex]

Step-by-step explanation:

[tex]y + \frac{3}{8} > 16 \\ y > 16 - \frac{3}{8} [/tex]

[tex]y > (16 \times 8) - ( \frac{3}{8} \times 8) [/tex]

[tex]y > 128 - 3[/tex]

[tex]y > 125[/tex]

Help pls! Will give brainliest to first person who answers correct

Answers

Answer:

a) x greater than or equal to 10

b) x = 9

Step-by-step explanation:

a)

[tex]x + 5 \geqslant 15 \\ x \geqslant 15 - 5 \\ x \geqslant 10[/tex]

b)

[tex]2x < 18 \\ x = \frac{18}{2} \\ x = 9[/tex]



Which statement BEST explains why the sine of an acute angle is equal to the cosine of the angles complement?

Answers

Answer:

In a right triangle, the sine of one acute angle, A, equals the cosine of the other acute angle, B. Since the measures of these acute angles of a right triangle add to 90º, we know these acute angles are complementary. ∠A is the complement of ∠B, and ∠B is the complement of ∠A.

Step-by-step explanation:

is there any answer choices i can see? :)

HELP ASAP PLZZZZZZZ If the side lengths of a cube are 15 feet, what is the correct way to write the expression to represent the volume of the cube in exponential form?

15 ⋅ 15 ⋅ 15
15 ⋅ 3
315
153

Answers

Answer:

D

Step-by-step explanation:

It's a guess because none of them look like the exponential form. However, I think D is probably the answer you want.

On this site it should be written as C = 15^3.

The correct anwser is D

A graph is shown. The x-axis is labeled from 0 to 9 and the y-axis is labeled from 0 to 10. The four points shown on the graph are ordered pairs 0, 0 and 1, 2 and 2, 4 and 3, 6. These points are joined by a line. The label on the x-axis is Bags of candy. The title on the y-axis is Total cost in dollars. What is the constant of proportionality?

Answers

Answer:

Heres the graph:

Step-by-step explanation:

Constant is $2 dollars per bag od candy or just 2.

Answer:

2

Step-by-step explanation:

from (0,0) ⇒ (1,2) ⇒ (2,4) ⇒ (3,6)

∴ 2

What is the value of x that makes L1 || L2? please help!! A. 18 B. 12 C. 10 D. 6

Answers

Answer:

x = 18

Step-by-step explanation:

Because the 2 lines are parallel we can use same side interior theorem to find  x

This would mean the equation is 6x + 30 + 2x + 6 = 180

When you solve that this would be 18

Answer:

x = 18

Step-by-step explanation:

→ Utilise the fact that interior angles add up to 180

6x + 30 + 2x + 6 = 180

→ Simplify

8x + 36 = 180

→ Minus 36 from both sides to isolate 8x

8x = 144

→ Divide both sides by 8 to isolate x

x = 18

Interpret the average rate of change of –14/3 that you found previously. What does this mean in terms of the waterslide, from x = 0 to x = 15?

Answers

Answer:

Solution: The average rate of change of  represents that the second variable is decrease by 14 if the first variable increase by 3. In terms of water slide the vertical height of slide is decreased at the rate of 4.66 per unit. The vertical height of the slide decrease 70 units from x = 0 to x = 15.

Explanation:

The rate of change shows the change in first variable with respect to change in second variable.

If the rate of chang is given by  it means second variable is decrease by 14 if the first variable increase by 3.

In the context of water slide it shows that the vertical height of slide is decreased by 14 units as we cover the distance of 3 units.

If we cover the distance from x = 0 to x = 15, it means we cover the distance of 15 units.

Let the vertical height of the slide covered from x = 0 to x =15 be y.

It means when distance increased by 15 units the height of slide decreased by y units.

So rate of change = .

It is given that the rate of change is .

Equate both equations.

Therefore, the average rate of change of  represents that the second variable is decrease by 14 if the first variable increase by 3. In terms of water slide the vertical height of slide is decreased at the rate of 4.66 per unit. The vertical height of the slide decrease 70 units from x = 0 to x = 15.

if
[tex] \sqrt{5} x = \sqrt{3} x + \sqrt{7} [/tex]
find the value of x in the form
[tex] \sqrt{ \frac{a}{b} } [/tex]

Answers

Answer:

[tex]$x=\sqrt{\frac{7(4+\sqrt{15})}{2}} $[/tex]

Step-by-step explanation:

From the way it is written, the [tex]x[/tex] is outside the square root. I will rewrite it as:

[tex]x\sqrt{5} =x\sqrt{3} +\sqrt{7}[/tex]

[tex]x\sqrt{5}-x\sqrt{3}=\sqrt{7}[/tex]

[tex]x(\sqrt{5} - \sqrt{3} )=\sqrt{7}[/tex]

[tex]$x= \frac{\sqrt{7} }{\sqrt{5} - \sqrt{3}} \implies \frac{\sqrt{7}(\sqrt{5} + \sqrt{3}) }{2} $[/tex]

[tex]$x=\frac{1}{2} \sqrt{7} (\sqrt{5} + \sqrt{3} )$[/tex]

[tex]$x=\frac{\sqrt{35}}{2} +\frac{ \sqrt{21}}{2} $[/tex]

[tex]$x=\frac{\sqrt{35}+\sqrt{21}}{2} $[/tex]

Multiply denominator and numerator by 3

[tex]$x=\frac{3\sqrt{35}+3 \sqrt{21}}{6} $[/tex]

Factor [tex]\sqrt{3}[/tex]

[tex]\sqrt{3} (\sqrt{105}+3 \sqrt{7})[/tex]

[tex]$x=\frac{\sqrt{3} (\sqrt{105}+3 \sqrt{7})}{6} $[/tex]

Divide denominator and numerator by [tex]\sqrt{3}[/tex]

[tex]$x=\frac{\sqrt{105}+3 \sqrt{7}}{2\sqrt{3} } $[/tex]

Let's rewrite it again

[tex]$x=\frac{\sqrt{ (\sqrt{105}+3 \sqrt{7})^2}}{\sqrt{12} } $[/tex]

[tex]$x=\sqrt{ \frac{1}{12} \cdot (\sqrt{105}+3 \sqrt{7})^2}$[/tex]

It is already in the form [tex]$\sqrt{\frac{a}{b} } $[/tex]

Expanding the perfect square, we have

[tex]63+42\sqrt{15}+105[/tex]

[tex]$\frac{63}{12} +\frac{42\sqrt{15}}{12} +\frac{105}{12} $[/tex]

[tex]$\frac{21}{4} +\frac{7\sqrt{15}}{2} +\frac{35}{4} $[/tex]

Factor [tex]$\frac{7}{2} $[/tex]

[tex]$\frac{7}{2} (4+\sqrt{15} )$[/tex]

Therefore,

[tex]$x=\sqrt{\frac{7}{2} \left(4+\sqrt{15} \right)} $[/tex]

[tex]$x=\sqrt{\frac{7(4+\sqrt{15})}{2}} $[/tex]

x= 7/4 x 25/6 is the answer

Tamara walked 3/4 mile in 1/2 hour. Which of the following represents the unit rate that Tamara walked? A. 1/2 mi/h B. 2/3 mi/h C. 3/4 mi/h D. 1 1/2 mi/h Include ALL work please!

Answers

Answer:

1 1/2 miles / hour

Step-by-step explanation:

We want distance / time

3/4 miles

---------------

1/2 hour

3/4 ÷ 1/2

Copy dot flip

3/4 * 2/1

3/2

1 1/2 miles / hour

Answer:

D

Step-by-step explanation:

Multiple (3/4)  by 2 to find the information for one hour)

6/4 in one hour

3/2 or 1 + 1/2 mi/h

D is the answer

help a girl out bc im strugglinggg also, these are two different questions, you can answer both or one of them thank you so much in advance

Answers

Step-by-step explanation:

yes the f( x) and g( x) is inverse each other.

f(x) = 3x-5

3x-5=0

x=5/3

g(x) = x+5/3

5x/3

x=3/5

yes
5/3
3/5
ik i’m right i took the test

8 w 4 + 10 w 3 − 12 w 2 Factor completely

Answers

Answer:

[tex] 2w^2(w + 2)(4w - 3) [/tex]

Step-by-step explanation:

[tex] 8w^4 + 10w^3 − 12w^2 = [/tex]

First, factor out the GCF of all terms, 2w^2:

[tex] = 2w^2(4w^2 + 5w - 6) [/tex]

Now we work on the trinomial.

Think of ax^2 + bx + c.

Multiply ac

ac = 4 * (-6) = -24

Now we need two numbers whose product is -24 and whose sum is 5.

They are -3 and 8.

We break up the middle term, 5w, into a sum using these two numbers.

5w = 8x - 3w

[tex] = 2w^2(4w^2 + 8w - 3w - 6) [/tex]

Now we factor by parts. We factor a common factor out of the first two terms, and we factor a common factor out of the last two terms.

[tex] = 2w^2[4w(w + 2) - 3(w + 2)] [/tex]

[tex] = 2w^2(w + 2)(4w - 3) [/tex]

Please help me! I will give brainly!

Asapppppp

Answers

Answer:

20

Step-by-step explanation:

I believe it's 20 because you multiply 20 . with 60 and 80 and it will give 1200 and 1800.

Answer:

It should be 20

Step-by-step explanation:

The question is asking multiply the serving by __ to find amount of pretzels.

Be careful what is the question is asking.

30*20=600

60*20=1200

90*20=1800

and continue on...

Pls give brainliest

Find RS. A. 5 B. 10 C. 9 D. 12

Answers

Answer:

x=9

Step-by-step explanation:

PQ + QR + RS = PS

2x-6 + 1 + x-4 = 18

Combine like terms

3x -9 = 18

Add 9 to each side

3x-9+9= 18-9

3x = 27

Divide by 3

3x/3 = 27/3

x = 9

Answer:

RS=5

Step-by-step explanation:

solve for x:

2x-6+1+x-4 = 18

combine like terms

3x-9=18

subtract both sides to make x isolated

3x=27

divide by 3 to get x

x=9

RS=x-4

RS=9-4

RS=5

Figure ABCD has verticies A(-3, 2) B(2, 2) C(2, -4) D(-3, -2). What was the area of Figure ABCD.

Answers

Okay so A is 5 units down from B and B is 6 unit horizontally away from C so 5 times 6 is 30 so the area of figure ABCD is 30 units
The answer is 30. Give the person above brainiest. This person explained it well

A 20% discount on pants, and its price became 48 after the sale, what was its price before​

Answers

Answer:

$60

Step-by-step explanation:

Represent the original price by p.  Then:  

1.00 p = original price

0.20 p = discount

0.80p = price after discount = $48

Solving for p, we divide both sides by 0.80:

        $48

p = ---------- = $60

        0.80

Fill in the blank with a constant, so that the resulting quadratic expression is the square of a binomial. $\[x^2 + 22x + \underline{~~~~}.\]$

Answers

Answer:

[tex]$\[x^2 + 22x + 121\]$[/tex]

Step-by-step explanation:

Given

[tex]$\[x^2 + 22x + \underline{~~~~}.\]$[/tex]

Required

Fill in the gap

Represent the blank with k

[tex]$\[x^2 + 22x + k\]$[/tex]

Solving for k...

To do this, we start by getting the coefficient of x

Coefficient of x = 22

Divide the coefficient by 2

[tex]Result = 22/2[/tex]

[tex]Result = 11[/tex]

Take the square of this result, to give k

[tex]k= 11^2[/tex]

[tex]k= 121[/tex]

Substitute 121 for k

[tex]$\[x^2 + 22x + 121\]$[/tex]

The expression can be factorized as follows;

[tex]x^2 + 11x + 11x + 121[/tex]

[tex]x(x + 11)+11(x+11)[/tex]

[tex](x+11)(x+11)[/tex]

[tex](x+11)^2[/tex]

Hence, the quadratic expression is [tex]$\[x^2 + 22x + 121\]$[/tex]

The answer would be x^2+22x+121

PLEASE HELP WILL GIVE BRAINLY!!!!!!!!!What is the vertex of the graph of the function f(x) = x2 + 8x − 2 ?
(−4, 18)
(0, -2)
(-8, -2)
(−4, −18)

Answers

Answer: D.  (-4, -18)

===================================================

Explanation:

A graph is a nice addition (and it will likely help us see the vertex directly), but it isn't necessary because we can use the equation given to us. Though I recommend using a graphing calculator to confirm the answer.

The original function is the same as y = 1x^2+8x + (-2). We see that it is in the form y = ax^2+bx+c where

a = 1

b = 8

c = -2

Use the values of 'a' and b to get the value of h, which is the x coordinate of the vertex.

h = -b/(2a)

h = -8/(2*1)

h = -4

The x coordinate of the vertex is x = -4. Plug this into the original equation to get

f(x) = x^2+8x-2

f(-4) = (-4)^2 + 8(-4) - 2

f(-4) = -18

Plugging x = -4 into f(x) leads to y = -18. The point (-4, -18) is on the parabola. Furthermore, this is the vertex (h,k)

------------

Alternatively, you can complete the square as shown below

y = x^2 + 8x - 2

y = (x^2 + 8x) - 2

y = (x^2 + 8x + 0) - 2

y = (x^2 + 8x + 16 - 16) - 2

y = (x^2 + 8x + 16) - 16 - 2

y = (x+4)^2 - 18

y = 1(x+4)^2 - 18

y = 1(x-(-4))^2 - 18

The last equation is in the form y = a(x-h)^2 + k with (h,k) = (-4,-18) being the vertex. The 16 is the result of taking half of 8 and squaring that result. We have 16-16 = 0 to make sure that we don't change the equation and keep things balanced. This is the same as adding 16 to both sides. All of this is done so we can end up with the (x+4)^2 perfect square portion.You can expand out (x+4)^2 - 18 and you should get x^2+8x-2 again.

D is the answer

Because you need to rewrite in vertex form and use this form to find the vertex (h,k)

Which should get you to

(-4,-18)

find the value of a answers: a:15 b:14 c:19 d:16

Answers

Answer:

6a+10= 3a+55

6a-3a= 55-10

3a= 45

a= 45/3

a= 15

The answer is (A) 15

Hope this helps^°^

Answer:

A. 15

Step-by-step explanation:

The angles are vertical angles, therefore they are congruent. We can set the two measures equal to each other.

[tex]6a+10=3a+55[/tex]

Now, solve for a. We must get a by itself on one side of the equation.

First, subtract 3a from both sides of the equation.

[tex]6a-3a+10=3a-3a+55[/tex]

[tex](6a-3a)+10=55[/tex]

[tex]3a+10=55[/tex]

Next, subtraction 10 to both sides of the equation. We subtract because 10 is being added to 3a. The inverse of addition is subtraction.

[tex]3a+10-10=55-10[/tex]

[tex]3a=55-10[/tex]

[tex]3a=45[/tex]

3 and a are being multiplied. The inverse of multiplication is division. Divide both sides of the equation by 3

[tex]3a/3=45/3[/tex]

[tex]a=45/3[/tex]

[tex]a=15[/tex]

The value of a is 15 and choice A is correct.

Two boxes are shown below

What is the difference in the surface areas, in square feet, of the two boxes?

Answers

Answer:

Option (B):3

Step-by-step explanation:

S.A of Box J =57ft^2 ; using formula 2(lb+lh+hb)

S.A of BoxF=54Ft^2;using formula (6a^2)

Differenceof S.A of boxes:57-54=3

C) 21 I’m not too sure but I think it’s C or B) 3

Use addition to solve the linear system of equations. 2x + y = 5 3x + 2y = 4 (6,7) (-6,-7) (6,-7) (-7,6)

Answers

Answer:

Solving 5x + y = 5 and - 3x + 2y = 6 gives (4/13,45/13) Similarly solve other combinations by observing graph to get other coordinates. From the figure we have obtained coordinates of corners as:

Step-by-step explanation:

The Solution for the linear equation is ( 6, -7)

What is Linear Equation?

First order equations include linear equations. In the coordinate system, the linear equations are defined for lines. A linear equation in one variable is one in which there is a homogeneous variable of degree 1 (i.e., only one variable). Multiple variables may be present in a linear equation.

Given:

2x + y = 5.......................(1)

and, 3x + 2y = 4....................(2)

Multiply the equation (1) by 2 we get

4x + 2y = 10 ....................(3)

Solving Equation (2) and (3), we get

  3x  +  2y  = 4      

  4x  +  2y  = 10

-         -           -

_____________

    -x = -6

      x= 6

and, y= 5- 2x = 5-2(6) = -7

Hence, the solution is (6, -7)

Learn more about linear equation here:

https://brainly.com/question/11897796

#SPJ2

In which of the following cases would the quantity 'q' decrease proportionally when the quantity 'p' is increased?
a) P = 3 x q
b) p-3=q
c) p=3/q
d) 1/p=3/q​
Please give me the correct answer along with the explanation and I shall mark you as the brainliest. Please answer ASAP, it's urgent

Answers

Answer:

c) p = 3/q

Step-by-step explanation:

If 3 is divided by a smaller number "q", then it would result in a larger number "p".

Example:

1. Substitute 3 for "q"

3/3 = 1

q = 3

p = 1

2. Substitute 1 for "q"

3/1 = 3

q = 1

p = 3

When "q" decreased, "p" increased.

Step-by-step explanation:

c) p=3/q

i can confirm after 10.30

I really need help with this

Answers

Answer:

8/5

Step-by-step explanation:

There are 8 jokers combined, and 5 decks, so the mean is 8/5.

8/5 explanation gut above me

if all the possible values of x are indicated by the shaded part of the number line above, which number line best shows all possible values of 1/x? (the shaded part in the number line is 0.5 to 1.5, the whole number line segment is from 0 to 3)

Answers

Answer:

can i see a picture of the number line?

Step-by-step explanation:

Do you have a picture of the number line?

Question 8(Multiple Choice Worth 1 points)
(06.05 MC)
A paper cup is dropped and its landing position is recorded. The cup can land on the side, on the open end, or on the closed end. The results of 20 trials are shown in the table below:
Paper Cup Experiment
# of times occurred
Open
HT III
Closed
Side
HT III
Based on the table, which of the following best compares the experimental probability of the cup landing on its open end with the experimental probability of the cup landing on its closed end?
The probabilities are equal.
The probability of landing on the open end is greater.
The probability of landing on the closed end is greater.
O No conclusion can be made.

Answers

Table Given in the question :

Open = HT 111 = 8

Side = 1111 = 4

Side = HT 111 = 8

Answer:

The probability of landing on the open end is greater.

Step-by-step explanation:

Given the experimental probability distribution :

Open = HT 111 = 8

Side = 1111 = 4

Side = HT 111 = 8

The experimental probability is the ratio of the number of times an event occurs and the total number of trials.

P(A) = number of times A occurs / total number of trials

Where A is defined event.

COMPARING the probabilities of open and closed events

P(open) = 8 / 20 = 2 / 5

P(closed) = 4 / 20 = 1/5

2/5 > 1/5

P(open) > P(closed)

2+2=4 yea yea that’s fine

Explanation/Answer would be appreciated please

Answers

Answer: The solution for the system is (2, -7)

Step-by-step explanation:

Ok, here we have linear relationships.

A linear relationship can be written as:

y = a*x + b

where a is the slope and b is the y-axis intercept.

For a line that passes through the points (x1, y1) and (x2, y2), the slope can be written as:

a = (y2 - y1)/(x2 - x1).

In this case, we have two lines:

ya, that passes through:

(-8, -5) and (-3, -6)

Then the slope is:

a = (-6 - (-5))/(-3 - (-8)) = (-6 + 5)/(-3 + 8) = -1/5

now, knowing one of the points like (-3, - 6) we can find the value of b.

y(x) = (-1/5)*x + b

y(-3) = -6 = (-1/5)*-3 + b

-6 = 3/5 + b

b = -6 - 3/5 = -33/5

then the first line is:

ya =  (-1/5)*x  -33/5

For the second line, we know that it passes through the points:

(-8, -15) and (-3, -11)

Then the slope is:

a = (-11 - (-15))/(-3 -(-8)) = (-11 + 15)/(-3 + 8) = 4/5

The our line is:

y(x) = (4/5)*x + b

and for b, we do the same as above, using one of the points, for example (-3, -11)

y(-3) = -11 = (4/5)*-3 + b

b = -11 + 12/5 = -(55 + 12)/5 = -43/5

then:

yb = (4/5)*x - 43/5.

Ok, our system of equations is:

ya = (-1/5)*x  -33/5

yb = (4/5)*x - 43/5.

To solve this, we suppose ya = yb

then:

(-1/5)*x + -33/5 = (4/5)*x - 43/5.

-33/5 + 43/5 = (4/5)*x + (1/5)*x

10/5 = 2 = (4/5 + 1/5)*x = x

2 = x

now we evaluate x = 2 in one of the lines:

ya = (-1/5)*2 -33/5 = -2/5 - 33/5 = -35/5 = -7

Then the lines intersect at the point (2, - 7), which is the solution for the system.

(2,7) is the answer

Find the measure of the remote exterior angle. m∠x=(4n−18)°m∠y=(n+9)°m∠z=(151−5n)° A. 71 B. 16 C. 100 D. 46

Answers

Answer: A.71

Step-by-step explanation: 1.set up the equation

2. 4n-18+n+9=151-5n

3.combine like terms ->  5n-9=151-5n

4.solve for n

5.10n=160

n=16

now you have to plug 16 in for n in order to get the remote exterior angle.

151-5(16)

151-80

71°

The sum of any two interior angle is equal to the third exterior angle. The value of n will be 16°. Then the correct option is B.

What is the triangle?

A triangle is a three-sided polygon with three angles. The angles of the triangle add up to 180 degrees.

The triangle having the interior angle x and y, and z is the exterior angle of triangle.

m∠x = (4n − 18)°, m∠y = (n + 9)°, and m∠z = (151 − 5n)°

Then the value of n will be

We know that the sum of any two interior angle is equal to the third exterior angle. Then we have

∠x + ∠y = ∠z

4n - 18 + n + 9 = 151 - 5n

10n = 160

n = 16°

Then the correct option is B.

More about the triangle link is given below.

https://brainly.com/question/25813512

#SPJ2

Ms. Canton went to the store to buy grapes. She bought four bunches of purple grapes. Each bunch had the same amount of grapes (x) She also bought two bunches of green grapes.Each bunch of green grapes had the same amount of grapes in them (y). Write an expression that will show how many bunches of grapes Ms. Canton purchased. *

Answers

Answer:
4x+2y

Explanation:
x represents the purple grapes in one bunch, while y represents the green grapes in one bunch.
Given: 4 bunches of purple grapes (x), 2 bunches of green grapes (y)
Combine to get the expression 4x +2y
Hope this helps
4x+2y because there’s four bags of grapes and two bunches of each and you don’t know the amount of grapes in each of them.

HELPP! QUICKKKK! Lol
Problem:
Last year I attended the Jurassic World Dinosaur Exhibit with my family and my friend and her family. My
family includes 2 adults (including myself) and my child we paid a total of $94. Her family includes 2
adults and 2 children she paid a total of $110. What was the admission price for an adult ticket and how
much was it for the admission price for a child ticket?
Let a represent adult ticket price
Let c represent child ticket price

Answers

Answer:

Child ticket = $16.

Step-by-step explanation:

Let adult tickets = a;

child ticket = c.

2a + c = 94 — eq no. 1,

2a + 2c = 110 — eq no. 2,

then,from eq 1,

c = 94 - 2a. —eq no. 3.

Substitute eq 3 into eq 2,

2a + 2(94 - 2a) = 110

2a - 4a = 110 - 188

-2a /-2 = -78 / -2

a = 39.

Substitute value of a into eq 3,

c = 94 - 2(39)

= 94 - 78

c = 16.

Thus, child ticket = $16.

Child= $16 and Adult= $39
Because, for the first family 39a+16c= $94 total for that family and just do the math for the other one :)
Other Questions
3Select the correct answer.The angle of depression between the top of a 100-foot cliff and a ship approaching the shore is 37.cliff top37100feet37shipdWhat is the approximate distance, d, between the bottom of the cliff and the ship?166.2 feetOB. 60.2 feet.75.4 feetOD.132.7 feetResetNext A sinusoidal voltage is displayed on an oscilloscope screen. The separation on the screen between two adjacent peaks is 5.5 divisions, and you notice that the sweep speed is set to 1 ms per division. What is the frequency of the sinusoidal voltage A bicycle has wheels that are 60 cm in diameter. What is the angular speed of these wheels when it is moving at 4.0 m/s A family that includes a mixture of parents and children unrelated by blood isalso described as a(n): A. Biological family.B. Multigenerational family C. Blended family D. Extended family Translate the following phrase into an algebraic expression using the variable m. Do not simplify,the cost of renting a car for one day and driving m miles if the rate is $39 per day plus 45 cents per mile What do you think has been the most important invention in history and why Cunto es 324 por 171 the speaker viewpoint is how the speaker looks at or_______ about The following data are accumulated by Lone Peak Inc. in evaluating two competing capital investment proposals: 3D Printer Truck Amount of investment $32,000 $40,000 Useful life 4 years 9 years Estimated residual value 0 0 Estimated total income over the useful life $3,520 $14,400 Determine the expected average rate of return for each proposal. If required, round your answers to one decimal place. 3D Printer 55 % Truck 8 % Whats the difference between rational and irrational numbers? PLEASE - Select the correct answer. A bicycle has a momentum of 36 kg m/s and a velocity of 4 m/s. What is the mass of the bicycle? 9 kg 32 kg 40 kg 144 kg How important is Forensic Chemistry in Criminal Investigation? If you answer correct I will mark brainliest Helppp thank you!!!!! what is the main idea in this passage? the roots of government PLEASE HELP I WILL GIVE BRAINLIEST!Find the value of x PLEASE HURRYanalyze the map below and answer the questions that follow on the map above what landform is located at oval C the sahel the Sahara Desert the Nile River Valley the Atlas Mountains please select the best answer from the choices provided A line passes through the point (12,-4) and is perpendicular to the line with the equation y=6x +3 the enemy won because of their ____ numberA. senior B. superior C. upper D. all are correct